APMA Exam

Pataasin ang iyong marka sa homework at exams ngayon gamit ang Quizwiz!

3.3. What are some of the typical fees that a potential investor in a hedge fund should be aware of before making an investment?

An annual management fee of is charged, typically in the area of 2%. Most hedge funds also have an incentive (performance) fee between 10% and 20% of the fund's profits. The term "2 and 20" would refer to a 2% management fee and 20% of profits going to the hedge fund manager. There can be a hurdle rate that the fund manager must surpass in order to earn a performance fee. There can also be a surrender fee when the fund is sold.

c. marked to the market

An investor's margin position is checked each day when all futures contracts are "marked to the market," which means that all profits and losses on contracts are credited and debited to each investor's account.

anchoring bias

Anchoring refers to holding certain beliefs and then not changing those beliefs even with new information.

1.5. Unsystematic risk can be diversified away by holding approximately how many securities in a portfolio?

Approximately 10-15 securities are required to diversify away unsystematic risk.

2.7. Why does the futures price converge with the spot price as the expiration date of the contract approaches?

As with put and call options, the value of a futures contract per unit can be worth only the value of the underlying commodity (spot price per unit) on the expiration date.

Jonathan Guyton's work identifies two confidence standard percentages: 99% and 95%. What are the initial withdrawal rate (IWR) maximum percentages based on the three different equity allocations?

At the 99% Confidence Standard, Max IWR is: -4.5%-4.6% with 50% equities -5.2%-5.3% with 65% equities -4.7%-5.6% with 80% equities At the 95% Confidence Standard, Max IWR is: -4.8% with 50% equities -5.5%-5.7% with 65% equities -5.6%-6.2% with 80% equities

Non investment grade for Moody's agency? Non investment grade for S&P agency?

Ba, B, Caa, Ca BB, B, CCC, CC, C, D

4.1. Identify three similarities between preferred stock and long-term bonds.

Both pay fixed returns, both are subject to interest rate risk and purchasing power risk, and the returns of both are paid before common stock dividends are paid. In addition, the process of valuing or pricing them is essentially the same.

Discuss the difference between a cash flow-based portfolio and an income-based portfolio.

Cash Flow: A cash flow portfolio is a total return portfolio designed to fund a retirement budget on a year-by-year basis. Money for the budget may come from many sources, including dividends, increases in an equity portfolio, rental receipts, and the liquidation of assets. When a portfolio is structured around cash flow, it is trying to balance income generation, while maximizing growth, within reasonable safety parameters, so as to provide the highest levels of inflation-adjusted funding for as long as needed. Income: Portfolios structured to provide income are primarily invested in income-producing investments such as bonds. There are two primary problems with an income portfolio. First, because the portfolio does not effectively respond to inflation, the purchasing power of the portfolio will start to erode. The second problem is related to the first. If an attempt is made to build an income portfolio that can effectively handle inflation, the retiree will have to accumulate significantly more money than with a cash flow portfolio.

2.4. Explain the difference between when income is reported under the cash and the accrual methods of accounting.

Cash income is recognized when it is actually or constructively received. Constructive receipt occurs when there is an unrestricted right to income that has not been reduced to the taxpayers' possession. Accrual income is recognized when earned, not necessarily when received.

1.16. Describe features common to all convertible bonds.

Convertible bonds permit the investor to exchange (convert) the bond into common stock under certain conditions. The conversion feature helps persuade investors to buy the firm's debt securities. Convertible bonds usually are subordinate to the firm's other debt, and they offer a lower yield than comparable nonconvertible debt because of the benefit of the conversion feature. Like other corporate bonds, convertible bonds usually are issued in $1,000 denominations, pay semiannual interest, and have a fixed maturity date. They have a call provision, which can be used by the issuer to force conversion.

How to compute Beta?

Correlation Coefficient * Standard Deviation (of stock) / Standard Deviation (of market)

2.12. Briefly describe covariance, and its importance in constructing a well-diversified portfolio.

Covariance measures how much two investments are related to each other; in other words, how much they move together or apart. In order to diversify we must add an asset that does not behave in the same manner as the asset we already have, otherwise we are just duplicating our current position. The lower the covariance, the lower the correlation between the two assets, and the more diversification we achieve. As we add additional assets to a portfolio, we want to make sure they are not highly correlated with the assets we already own. However, covariance can be just about any number, so it can be difficult to understand the correlation.

a. current yield

Current yield is calculated by dividing the annual interest payment by the current market price. Current yield does not take into account the difference between a bond's purchase price and its redemption value.

a. If investors expect inflation to increase over the next several years, how will the yield curve change, and why?

In general, all interest rates along the curve will rise. Initially, long-term interest rates may increase more sharply than short-term rates. Investors who fear increased inflation will expect a higher current yield for bonds with longer maturities, which will compensate them for loaning money for an extended period of time. The yield curve should remain positively sloped. If the perception among investors regarding inflation continues, then short-term rates will also rise. At some point, short-term rates may rise faster than long-term rates, especially if the Fed intervenes.

intrinsic value

Intrinsic value, which is the underlying or inherent value of a stock, is a function of the stock's current dividend, the anticipated growth rate in dividends, and the investor's required rate of return.

Leveraged buyouts (LBOs)

LBOs occur when a publicly traded company (which is typically undervalued) is taken private. A common LBO scenario is to take the company private in order to repackage and restructure the company, and then return the company to the public market at a higher price through an IPO. Another LBO approach would be to break up the company into several parts and sell off the different parts of the business to different bidders. This process is referred to as "cut and run" and can result in mass layoffs, especially for the unprofitable parts of the company. An advantage to corporate management and investors with LBOs is that they do not have to answer to public shareholders. Since established and more mature companies are typical LBO candidates, there is less risk with LBOs than with venture capital.

Jensen's alpha measures the risk premium in terms of beta. On what assumption is this risk measure based?

Measuring the risk premium in terms of beta is based on an assumption that the portfolio is well diversified. If a portfolio were not sufficiently diversified, the portfolio's risk would include both unsystematic and systematic risk, and the standard deviation of the portfolio's returns would be a more appropriate measure of risk.

mental account bias

Mental accounting is when individuals divide their assets into different pockets. A gift of $500 (which may be treated as found money) is not treated the same as $500 of earnings, but in each case the individual has the same amount. Mental accounting also occurs when an investor treats each of their accounts separately rather than looking at them as part of their overall portfolio.

Mezzanine financing

Mezzanine financing is essentially debt with an equity component. Mezzanine debt gets its name from the fact that it is between common equity (the floor) and bank loans and senior debt (the ceiling). Mezzanine financing is often found in the middle market—companies that are not yet large enough to have ready access to the capital markets but are larger than venture capital firms.

4.2. How is modeling used in the investment selection process?

Modeling is used to develop an optimistic, pessimistic, and most probable outcome for a base scenario, and helps an analyst develop a more realistic analysis in conditions of uncertainty. Modeling is used for many purposes, including enabling the presence of economic forecasts in capital market analysis and retirement planning in financial planning engagements.

2.10. What differentiates a mortgage REIT from an equity REIT?

Mortgage REITs invest in loans that finance real estate purchases. They also may invest in Ginnie Maes or other mortgage-backed securities. Mortgage REITs generally provide higher current income with less opportunity for capital gains than do equity REITs. Equity REITs own real estate properties and receive income from the rental or lease of these properties. Equity REITs provide income and capital appreciation because the trust actually owns property.

2.10. Explain the taxation of futures contracts.

Net profits on futures trades are taxed as capital gains (60% long term and 40% short term). All positions (including open positions) are considered closed at the end of the year and are taxed in that year.

2.3. What are some benefits and risks of asset-backed securities (ABSs)?

New issues of ABSs typically have higher yields than many corporate bonds of comparable maturities and credit quality. There is prepayment and reinvestment risk, and there is a wide variation in the liquidity and marketability of ABSs. Securitization allows institutions to remove loans from their balance sheets. Investors need to be aware of credit risk and the possibility that a loan may not be repaid.

4.1. What are exchange-traded funds (ETFs)?

Exchange-traded funds are open-end investment companies that trade throughout the day, almost all of which trade on the American Stock Exchange. More specifically, ETFs are baskets of securities that often track stock market indexes (both U.S. and international), specific sectors of the stock market, or in a few cases, bond indexes. As the popularity of ETFs has grown, more specialized ETFs have been developed and offered. Therefore, an investor can build a portfolio of core ETFs and fine-tune it with a wide variety of different investment options. Expense ratios between 9 and 20 bps; specialized over 50 bps.

framing bias

Framing is how something is presented to an individual, such as buying a "$50" shirt for $35. With investments, individuals will often choose a guaranteed positive outcome (while avoiding a chance of a greater gain that also carries the possibility of no gain at all), but they will take a chance to avoid a negative outcome (rather than take a certain smaller loss).

the asset management process

G.E.A.R.M. 1. Gather date 2. Establish goals 3. Analyze 4. Recommendations/Implementations 5. Monitor

the "guardian"

Guardians are highly anxious about the safety of their money; typical clients in this category are retired investors.

hindsight bias

Hindsight is 20-20 and, looking back, events seem more predictable than they were. If a financial professional recommends an investment and it turns out well, clients tend to think they liked it from the start. However, if an investment turns out poorly, clients think they had doubts with the investment to begin with

4.1. The capital asset pricing model is the basis for the Jensen, Treynor, and Sharpe performance measurements. What does the CAPM specify about the required return on an investment?

The CAPM specifies the required return on an investment depends on (1) the return an investor may earn on a risk-free asset and (2) a risk premium. This is important in performance measurement because all three approaches measure whether the portfolio is achieving the risk premium anticipated.

Commodity Futures Modernization Act of 2000

The Commodity Futures Modernization Act essentially exempted derivatives such as credit default swaps from regulation by the CFTC or any other government regulatory agency. This led to an explosion in the notional (face value) of derivatives, which contributed heavily to the subprime mortgage meltdown and financial crisis in 2008

b. If the Fed also believes that inflation will increase, how will the yield curve change, and why?

The Fed can control short-term rates, not long-term rates and will probably raise the Fed funds rate and the discount rate, which would force up short-term interest rates. This might cause short-term rates to rise more rapidly than long-term rates, which would cause the yield curve to flatten and possibly to become a negatively sloped curve.

Investment Advisers Act of 1940

The Investment Advisers Act of 1940 wrote into law the fiduciary duty owed by investment advisers to their clients. This act requires investment advisers to register with the SEC by filing Form ADV. This act also contains certain prohibitions regarding advertising practices and certain requirements for disclosure. Currently, investment advisers with over $100 million AUM register with the SEC, and all other investment advisers register at the state level.

Investment Company Act of 1940

The Investment Company Act of 1940 subjects both the structure and the operations of mutual funds to detailed regulation. Mutual funds belong to a class of investment companies defined in the 1940 act as "management companies," which are further classified as either diversified or nondiversified. The act also requires mutual funds to maintain detailed books and records on the securities owned, to use a custodian to safeguard the securities, and to send semiannual and annual reports to both the SEC and shareholders. The act also requires that proceeds from redeemed shares be sent to shareholders within seven days of the redemption.

2.9. What does semivariance measure?

Semivariance measures only the returns that fall below the average, and is primarily used by portfolio managers. This measure recognizes that investors are concerned less about upside potential and more about downside risk.

4.1. What is sensitivity analysis?

Sensitivity analysis is used to evaluate the risk associated with a given investment and assesses the impact of different variables on an investment's returns.

1.3. What is the alternative minimum tax (AMT)?

The alternative minimum tax is a tax imposed on individuals who use various deductions, exemptions, and tax sheltering mechanisms as a way of making those individuals pay a minimum amount of tax. Certain tax preference or adjustment items, such as tax-exempt interest on qualified private-activity municipal bonds (except those issued in 2009 and 2010), depletion allowances and intangible drilling costs from oil and gas activities, are adjustments (increases) to taxable income. The alternative minimum taxable income is taxed at either a 26% or 28% rate.

Explain the tax consequences of an annuity payment.

The annuity payment is comprised of taxable interest income and nontaxable return of capital.

3.7. The standard deviation of the return on the market index is estimated as 20%. If Asset A's standard deviation is 30% and its correlation of returns with the market index is 0.8, what is Asset A's beta?

1.2

2.13. The standard deviation of the market is 14, and the standard deviation of United Enterprises Inc. is 22. The correlation coefficient between the two is .85. What is their covariance?

261.80

1.16. Your client purchased a bond for $950. The bond has a coupon rate of 3%, matures in 17 years, and is callable in five years at $1,015. What is the YTC for this bond?

4.40%

Investment grade for Moody's agency? Investment grade for S&P agency?

Aaa, Aa, A, Baa AAA, AA, A, BBB

What is the range of safe withdrawal rates suggested by Bengen's layer cake study?

Depending on the individual's response to the seven factors, the safe initial withdrawal rate ranges from a base rate of 4.15% to a maximum rate of 7%. More conservative individuals might want to begin at the 4% level.

Distressed debt

Distressed debt is investing in the debt of companies that are in trouble or failing. As with mezzanine debt, distressed debt is tied less to the performance of the overall market and more toward the individual circumstances of the company itself. The main risk with distressed debt is business risk, namely that the company could go out of business completely. In many ways, distressed debt investors are actually equity investors in that they are betting on the financial survival of the company, and they will most likely have a say in how the company is restructured. This restructuring could include converting debt into equity ownership for the distressed debt holders.

Dodd-Frank Wall Street Reform and Consumer Protection Act

Dodd-Frank addressed the issue of "too big to fail" and required that banks met certain capital requirements and went through stress tests to ensure they would be able to survive an unexpected shock to the financial system. Dodd-Frank also regulated derivatives. The act shifted derivatives trading from the over-the-counter (OTC) market to central clearing counterparties (CCPs) that facilitate the netting of swap contracts in order to reduce systematic risk. There are also investor protections in the bill, and the Consumer Financial Protection Board (CFPB) was established.

2.4. What is duration, and how is it used?

Duration tells an investor the approximate price movement of a bond (or a bond mutual fund) given a 1% change in interest rates. Duration is the weighted-average amount of time it takes to collect a bond's interest and principal payments. Duration is used to compare the interest rate risk of bonds that have different coupons and different maturities (i.e., to relate bond price sensitivity to interest rate changes). Investors can reduce interest rate risk by selecting bonds with shorter durations. They also can match the duration of their portfolios with the timing of their cash flow needs. By matching duration to the term of a goal, they optimize the trade-off between interest rate risk and reinvestment risk.

1.4. Explain the significance of endogenous risk.

Endogenous risk is a risk found within the financial system, and occurs when there is a shock (or panic), which then spreads and is amplified within the system.

What are the 6 asset classes?

Small-cap stocks Large-cap stocks Long-term government bonds Long-term corporate bonds Intermediate-term government bonds U.S. Treasury bills

the "individualist"

The individualist approaches life with confidence and exercises care in reaching important decisions; typically, the small business owner or professional falls under this category.

Explain why investing in tax-exempt bonds may not be a good tax strategy for Social Security recipients.

The interest from tax-exempt bonds increases the taxpayers' "provisional income." If provisional income exceeds certain levels, then up to 85% of the Social Security benefit may be taxable. Therefore, tax-exempt interest might cause a person's Social Security benefits to be taxed.

Explain the tax treatment of U.S. EE savings bonds.

The interest income from EE bonds is deferred until redemption or maturity. However, the taxpayer may elect to currently include the accrued interest in income on an annual basis.

c. What is the bond's investment premium?

The investment premium is $294.21, the difference between the bond's market price of $1,200 and the bond's investment value of $905.79.

1.7. An investor currently holds $25 put options on a stock that is trading at $22 per share. She does not anticipate the stock changing in price between now and expiration. The $25 put option is currently trading at $4.50. What action should the investor take?

The investor should consider selling the put options. Currently there is a $3 intrinsic value ($25 - $22), so the remaining $1.50 is a time premium ($4.50 - $3.00 intrinsic value = $1.50 time premium). As the expiration date approaches, the time premium will disappear, leaving only intrinsic value at expiration. If the investor does not anticipate the stock changing in price between now and expiration, she could capture the $1.50 time premium if she sells now. If she waits until expiration, only intrinsic value will be left.

a. liquidity preference theory

The liquidity preference theory argues that longer-term securities have higher interest rates because they have more risk and because investors have to be compensated for giving up access to their funds for a longer period of time.

2.1. What is the difference between the marginal and the average income tax rates?

The marginal income tax rate applies to the last dollar of taxable income. The average income tax rate is the income tax liability divided by taxable income. The average rate will always be less than the marginal rate.

1.10. Explain two major differences between the Treasury securities in the previous question and Treasury Inflation-Protection Securities (TIPS).

The principal amount of the TIPS changes at each interest payment period in step with the increase or decrease in inflation since the last payment period. The TIPS coupon rate is then applied to the adjusted principal amount to determine the interest payment amount.

What is a pro forma cash flow statement?

The pro forma cash flow statement is a planning tool that projects the anticipated amounts and timing of inflows and outflows for a future period.

b. semi-strong form

The semi-strong form of the EMH asserts that all publicly known and available information is fully reflected in stock prices. This suggests that neither technical analysis nor fundamental analysis can produce superior results over time on a risk-adjusted basis. Only private or inside information can produce superior results.

2.2. What is the spot price of a commodity? What is the futures price?

The spot price of a commodity is its current price in the cash market. The futures price is the price specified in a contract for future delivery of the commodity.

What is a statement of financial position?

The statement of financial position is a snapshot of an individual's assets, liabilities, and net worth as of a given date.

c. strong form

The strong form of the EMH asserts that stock prices fully reflect all information, both public and private. Not even access to inside information can be expected to result in superior investment performance over time.

Securities Investor Protection Act of 1970

This act established the Securities Investor Protection Corporation (SIPC) to oversee the liquidation of brokerage firms and to insure investors' accounts up to a maximum value of $500,000 (of which only up to $250,000 can be cash balances) in the case of bankruptcy of a brokerage firm. This corporation was set up in response to the problems encountered by brokerage firms in the late 1960s, when high volume caused a back-office paper crunch leading to bankruptcies and subsequent investor losses. While the SIPC insures brokerage accounts, it does not cover market losses suffered while waiting to get securities from a bankrupt brokerage firm. It also does not cover losses due to investment fraud and should not be thought of as the securities world equivalent of the Federal Deposit Insurance Corporation. The cost of this insurance is paid by members of SIPC. All brokers and dealers that are registered with the SEC and all members of national securities exchanges must be members of SIPC.

Gramm-Leach-Bliley Act of 1999

This act is also known as the Financial Services Modernization Act, and it deals with ways that financial institutions handle the private information of individuals. The act is best known for repealing part of the Glass-Steagall Act of 1933, which prohibited financial institutions from consolidating and offering any combination of traditional commercial banking, investment banking (brokerage firms), and insurance.

USA Patriot Act

This act requires broker-dealers, among others, to have internal policies, procedures, and controls meet the "know your customer" mandate to combat terrorism and money laundering. Broker-dealers, in turn, may well ask their investment advisers to provide more detailed information about their clients. Advisers should look for red flags such as transactions that do not make sense for a client, numerous accounts held in different names or corporations for no apparent reason, and clients' lack of concern about investment objectives, risks, and investment costs. The adviser's requirements in complying with the act are still evolving, but the adviser should be aware of this act and its possible implications, including having formal policies in place to fight money laundering.

1.4. What is the kiddie tax?

Under the "kiddie tax" rules, the standard deduction is limited. For children subject to the kiddie tax, the first $1,100 of unearned income is sheltered by the child's limited standard deduction; the next $1,100 is taxed to the child at the child's tax rate. Any unearned income in excess of the first $2,200 is taxed to the child using the parent's marginal tax rate. The kiddie tax applies to children under age 18. The kiddie tax also applies to children age 18 and full-time students who are 19, but under the age of 24, provided the child's earned income does not exceed one-half of his or her support.

3.2. Explain how CMOs alter the payment pattern and reinvestment risk association with traditional mortgage-backed securities.

Unlike traditional pass-throughs, CMOs use a sequential distribution process that creates a series of bonds (tranches) with varying maturities. This process smooths out the payment pattern. The risk of prepayment still exists, but it is transferred to companion tranches associated with the issue.

Venture capital

Venture capital is the best-known type of private equity. Venture capital investors are seeking a high rate of return, but are also taking on a very high level of risk. Venture capital returns are tied to market performance, meaning venture capital performs best when the public stock markets are also doing well.

overconfidence bias

We all overestimate our abilities, and with investments it can lead to decisions based upon our limited knowledge. There are often many factors about an investment that the investor knows little or nothing about, and has no control over. In bull markets, investors often credit themselves for returns based on what they believe are their smart decisions and savvy stock-picking ability. Overconfidence is strongest in male investors.

1.8. How can investors use puts to minimize losses on specific stocks held in a portfolio?

When stocks are held in a portfolio and options on those stocks are available on an options exchange, puts can be purchased on all of the stocks in the portfolio, or just on selected stocks to minimize the potential loss on the stock if the market price declines. This strategy can be an expensive form of insurance protection if the put options are continuously maintained. However, if an investor is concerned (but unsure) about the price of a stock declining soon, a protective put can be beneficial. The investor can continue to hold the stock as a long-term investment, but can protect against a short-term decline by purchasing a put option on the stock.

a. What sort of characteristics would you look for in a bond chosen for a client with a high risk tolerance?

a bond with a high duration, a low or zero coupon, a long maturity, and a relatively low credit rating

c. What sort of characteristics would you look for in a bond chosen for a client with a low risk tolerance?

a bond with a low duration and a short maturity, a coupon that is at current market rates, and a high investment-quality rating

discontinuous change

a radical departure from a current trend or pattern

incremental change

a small and natural extension of an existing pattern that bears watching

3.9. An investor's required return is 10%. Stock CMR sells for $26 per share and pays an annual dividend of $.85; its dividends are expected to increase by 7% annually. CMR's earnings per share are $1.40, its sales per share are $29, and its book value is $14 per share. Compute the following financial statistics for stock CMR. a. intrinsic value using the DDM b. expected return c. P/E ratio d. PSR e. P/B ratio

a. 30.32 b. 10.50 c. 18.57 d. 0.90 e. 1.86

Define four basic purposes of the investment policy statement.

a. Setting objectives. b. Defining the asset allocation policy. c. Establishing management procedures. d. Determining communication procedures.

3.3. Beta coefficients may be used to help select a portfolio that mirrors an investor's willingness to _______________. The beta for individual stocks ________ over time. Beta coefficients for individual securities can be very _________. In a portfolio, some individual stocks' betas will rise, and some will fall. However, over time, a portfolio's beta can be _____________, as beta increases offset beta decreases. Therefore, a portfolio's average beta can be a good indicator of how ____________ a portfolio is likely to be in relation to the __________.

bear systematic risk changes unstable relatively stable volatile market

d. What bond characteristics would you search for if you believed that interest rates would decline sharply in the future?

bonds that have long maturities and high durations and that have low (or zero) coupons

e. What bond characteristics would you search for if you believed that interest rates would rise sharply in the future?

bonds that have short maturities and low durations and that have high coupon rates

3.4. Under the dividend growth model, what are the three factors on which a stock's intrinsic value is based?

current dividend expected growth rate required rate of return

1.1. Investors in U.S. bonds are subject to what three types of systematic risk?

interest rate risk reinvestment risk purchasing power risk

4.9. List the four attributes of preferred indexes.

relevant and appropriate comprehensive and broad-based investable and capable of being replicated value-weighted (capitalization weighted)

3.5. When a portfolio is not ____________________, beta may ______________ the risk borne by an investor because _______________ has not been eliminated from the portfolio

well-diversified understate unsystematic risk

What is a cash flow statement?

The cash flow statement is a statement that reveals an individual's cash receipts and disbursements over a specific period of time in the past.

2.20. What does the coefficient of determination (R2) tell us about systematic and unsystematic risk?

The coefficient of determination (R2) tells us how much of the risk is explained by the benchmark. For example, if we are comparing the ABC Fund to the S&P 500, and the coefficient of determination is .70, this would mean that 70% of the price movement of the ABC Fund is explained by the S&P 500 (systematic risk), and the other 30% is not explained by the S&P 500 Index (unsystematic risk). For diversification purposes, the lower the coefficient of determination (and thus the correlation coefficient) the better. However, beta is a measure of systematic risk, and because the coefficient of determination (R2) is a measure of systematic risk, when using beta the higher the coefficient of determination (R2), the better.

expected return

The expected return is the anticipated return on an investment and is the sum of interest or dividend income and capital gains.

4.1. What are the main expenses associated with investing in private equity?

The fees associated with private equity investing are similar to hedge fund fees; namely there is typically a management fee in the 2% to 2.5% range, and also incentive (performance) fees in the range of 20%-35% of any profits made. The management fee is charged on the amount of committed capital even if the amount of capital actually raised is less.

b. How would the company's stock price be affected if the company raises the payout ratio?

The price could rise or fall; we cannot determine the impact on the stock based solely on this decision.

b. pure expectations theory

The pure expectations theory argues that the current long-term rate is an average of the current short-term rate and expected future short-term rates. Thus, long-term yields are a function of the expected short-term rates that will prevail during the term of the long-term security. This theory, like the liquidity preference theory, supports an upward-sloping (normal) yield curve when short-term rates are expected to increase in the future. On the other hand, this theory supports a downward-sloping (inverted) yield curve when short-term rates are expected to decrease in the future.

4.4. What is the purpose of portfolio management?

The purpose of portfolio management is to match appropriate investment vehicles to your client's goals and time horizon while simultaneously considering your client's risk tolerance.

2.2. Explain the random walk hypothesis.

The random walk hypothesis holds that price movements are unpredictable and that a price movement today is unrelated to a price change yesterday or any other day.

required return

The required return is the minimum return the investor wants to receive to compensate for the risk associated with investing in a particular security.

risk-free return

The risk-free return is the nominal rate of return that an investor could earn on a risk-free security (e.g., U.S. Treasury bill).

Securities Acts Amendments of 1975 and May Day

This act directed the SEC to supervise the development of a national securities market. The assumption behind this act was that any national market would extensively use computers and electronic communications. This act also prohibited fixed commissions on public transactions, which fosters greater competition and more efficient prices. May Day is the name given the to the day in 1975 when commission rates were no longer set by the NYSE and firms were free to set their own commission rates.

c. yield-to-call

YTC is a measure of the yield for bonds that are likely to be called. In calculating YTC, the number of periods until the call date is used instead of the number of periods until maturity, and the call price is used instead of the face value.

2.1. What are some examples of asset-backed securities (ABSs)?

automobile loans, credit card receivables, home equity loans, home improvement loans, trade receivables, small business loans, student loans, and equipment leasing programs

hot broker clientele

listen to "hot" stories and tips; want to be doing what "everyone else" is doing; often buy high and sell low; herd mentality investment professional: counsel these clients against their instinct to run with the herd

taxation of cash (ordinary) dividend

long-term capital gain rates.

2.3. A ___________________ curve assumes that investment returns are normally distributed around the mean return. Most investment returns are _______________, meaning there are generally more positive returns and fewer negative returns; __________ occurs when there are a lot of returns clustered around the mean return with few large surprises.

normally distributed; positively skewed; leptokurtic kurtosis

2.15. A correlation coefficient of +1 means that the two assets are ______________________________, a correlation coefficient of 0 means ________________________________, and a correlation coefficient of -1 means that the two assets are _______________________________.

perfectly positively correlated there is no correlation perfectly negatively correlated

taxation of stock dividend

stock dividend does not constitute a taxable event, but it does dilute the basis of the shares held and establish basis in the newly acquired shares.

do-it-yourselfers

tend to handle their own financial affairs; not usually "sold" on products; will listen to a presentation of the available facts, do their own analysis, and then make their own decisions investment professional: act as a provider of information and as a sounding board

What is the basic principle in determining the fit of an investment recommendation?

to avoid redundancies and conflicts with current holdings

1.3. ____________________ can be diversified away by holding a mix of fixed-income and equity securities from different companies, industries, and countries.

unsystematic

1.2. __________ risk is diversifiable; ____________ risk is nondiversifiable.

unsystematic; systematic

a. What is a unit investment trust?

A UIT holds a fixed portfolio of securities; UITs are formed by brokerage firms and sold to investors in units of $1,000. UITs typically consist of a fixed portfolio of federal government bonds, municipal bonds, corporate bonds, or mortgage-backed securities such as GNMAs.

b. management fees

A management fee, which is paid out of a fund's income, is the amount paid by the fund to an investment adviser for its services.

a. sales charges (loads)

A sales charge, called a "load," is a charge for selling the fund to an investor. A front-end load applies the sales charge at the time of purchase. A back-end load, also called a continent deferred sales charge, is applied if the shares are sold within a certain period and is on a sliding scale. For example, if shares are redeemed within the first year of purchase, the charge might be 6%, 5% for redemptions within the second year, and so on until the charge disappears. A level-load is one whose sales charge is asset based; it uses the annual 12b-1 fee as a sales charge.

1.6. What relationships emerge between a call option's value and the variables listed in the previous question?

-An increase in the price of the stock increases the value of a call option. -An increase in the length of time until expiration increases the value of an option; as an option approaches expiration, its value decreases. -An increase in the volatility of the stock increases the value of an option (its speculative premium increases). -An increase in interest rates increases the value of an option. -An increase in the strike price decreases the value of a call option (because the intrinsic value of the option drops for a given stock price).

Capital Asset Pricing Model (CAPM)

-Assumes only market return and beta are necessary to explain all stock returns -Straight line relationship b/w risk and return -Stock beta is the only factor affecting returns

3.10. Summarize the relationships between price, coupon, maturity, interest rates, and duration.

-Bond prices and interest rate changes are inversely related. -Long-term bonds are more affected by interest rate changes than are short-term bonds. -Lower-coupon bonds are more affected by interest rate changes than are higher-coupon bonds. -Lower-rated bonds have more price volatility than higher-rated bonds. -Bonds with longer durations are more volatile than bonds with shorter durations. -There is a positive relationship between maturity and duration. -There is an inverse relationship between the market interest rate (YTM) and duration. -There is an inverse relationship between coupon rate and duration.

4.3. Constructing a systematically developed portfolio requires paying attention to which two factors?

-Eliminating unsystematic risk. -Minimizing the total risk, which is measured by standard deviation.

Security market line (SML)

-Micro aspect of capital market theory -Specifies the risk/return relationship for an individual security or for a portfolio of securities -Beta used as a measure of risk

Arbitrage Pricing Theory

-Multivariate model that recognizes variables other than market return and beta can affect stock prices -Non straight-line relationship b/w risk and return -Factors affecting returns include sector influences and systematic influences

5.7. What are the three primary factors in pricing contingent convertible bonds (CoCos)?

-the level of subordination (where the bond stands in the capital structure of the bank) -the defined trigger mechanism (can be rules based or discretionary) -the defined loss absorption mechanism (can include cancellation of coupon payments, principal write-down, or conversion into equity).

2.14. The covariance between Twin Pines Inc. and the S&P 500 is 95. The standard deviation of Twin Pines is 13, and the market's standard deviation is 12. What is their correlation coefficient?

0.6090

3.3. What are the three investment styles of private real estate funds?

1. Core - conservative 2. Value-added - moderate 3. Opportunistic - aggressive

5.1. What is a structured product?

A structured product uses a combination of derivatives and financial instruments to create a structure that meets a highly customized risk-return objective. Structured products are based or can be linked to a variety of different assets, including a single security, a basket of securities, an index, various commodities or currencies, or debt.

1.22. What are the expected effects of a weakening or strengthening U.S. dollar on the value of international funds?

A weaker U.S. dollar will increase the value of international funds for a U.S. investor, while a stronger U.S. dollar will decrease the value of these funds for a U.S. investor.

3.8; b. What is the intrinsic value of stock BLQ?

$67.77

What are the 4 major types of real estate?

1. Land - timberland, raw 2. Residential - homes, apartments, condos 3. Commercial - businesses, retail stores, hotels 4. Industrial - factories, warehouses, plants

List and explain the areas that are considered critical to a successful client/adviser relationship and how incorporating behavioral finance can enhance the relationship.

1. The adviser understands the long-term financial goals of the client. Behavioral finance helps the adviser understand the reasons for the client's goals, making the client feel like they are better understood. 2. The adviser maintains a consistent approach with the client. Behavioral finance adds structure and professionalism to the relationship, which helps the adviser understand the client before investment advice is given. 3. The adviser invests as the client expects. Once the adviser understands the motivations for the client's goals, the adviser is better equipped to meet the client's expectations. 4. Both client and adviser benefit from the relationship. The primary benefit of incorporating behavioral finance into the client/adviser relationship is a closer bond between them, resulting in happier clients and an enhanced practice for the adviser.

Jane Roberts owns a public purpose municipal junk bond that pays 6%. b. What if Jane itemizes deductions and is also in an 8% state marginal tax bracket? What yield on corporate bonds would then be comparable to Jane's current investment?

10.03%

2.5. Calculate the duration and expected price change for each of the following bonds. b. Coupon rate greater than market rate. Assume that the coupon is 8%, that the current market interest rate is 6%, that there are 22 years until maturity, and that compounding is semiannual. Also assume that interest rates are subsequently expected to rise by 60 basis points.

11.54 -0.0672 $83.53 drop in price

3.8; c. What is the expected return on stock BLQ?

11.72%

2.1. Christa Pate owns unimproved land with a current market value of $175,000, 50 NLR convertible bonds with a current market value of $48,500, and 1,000 shares of MPT stock with a current market value of $72,500. Christa expects returns of 14%, 21%, and 9%, respectively, on her investments. What is the overall weighted-average expected return on Christa's portfolio? (Set your calculator to four decimal places. Also, use the calculator function keys to solve the problem more quickly.)

13.92%

3.6. An investor is considering purchasing a $2.50 series A preferred stock. His required return is 10%. Should he purchase this stock if it is selling for $27 per share?

2.50/10% = 25 max price (overvalued)

1.10. Paulette Doyle's marginal tax bracket is 35%. She is considering either a corporate bond that pays 5.5% annually or a tax-exempt municipal bond. What yield on the municipal bond would be comparable to the yield on the taxable corporate bond?

3.58%

1.14. A bond has a market price of $950. The bond pays 4% coupon interest semiannually. The bond will mature in seven years and will pay a face value of $1,000. a. What is the YTM (IRR) for this bond?

4.85%

1.8. What are the advantages and disadvantages of investing in natural resources?

Advantages include the potential return associated with inelasticity of supply (i.e., the fixed supply), the diversification of a portfolio (i.e., nonfinancial assets versus financial assets), the long-term asset growth potential, and the potential tax advantages. Disadvantages include the negative side of a fixed supply (i.e., the existing supply does not disappear when demand falls, causing the price to be depressed for a longer time), the active management required when investing directly, and the possibility of political upheaval or natural disaster.

3.14. Kent Walters, age 32, has $40,000 to invest in a fixed-income security. He has invested in various types of bonds for 10 years, he considers himself to be an aggressive investor, and he is in the 24% marginal income tax bracket. His primary goal is capital appreciation; income is a secondary consideration. Kent's financial planner has presented the following securities and their before-tax yields. Investment A: 15-year, BB rated, noncallable corporate bonds trading near par with a yield of 6.8% Investment B: 20-year, A rated, discount, public purpose, callable general obligation municipal bonds with a taxable-equivalent yield of 7.2% Investment C: 10-year, A rated, premium, callable, sinking fund, corporate bonds with a yield of 4.5% Investment D: Treasury bills with a yield of 2.5% Which one of these fixed-income securities would be an appropriate choice for Kent, and why?

Investment B is an appropriate choice. On an after-tax basis, it has the highest return after-tax (5.47%), and when compared to the BB rated bonds with a 5.17% after-tax return, the municipal bonds have a higher after-tax yield with a better risk rating. Compared to the A rated corporate bonds, the municipal bonds are less likely to be called since they are trading at a discount. There is no reason for the investor to seek the security of Treasuries (with a 1.90% after-tax return), given his aggressive risk profile. Also, because his primary goal is capital appreciation, the discounted, 20-year bond is most likely to provide capital gains if interest rates decrease.

2.4. What requirements must REITs meet concerning investments, gross income, and earnings?

REITs must invest at least 75% of total assets in real estate, and must derive at least 75% of gross income from rents. Also, at least 90% of earnings must be distributed to shareholders.

3.1. Explain the characteristics of public nontraded REITs, and how they differ from publicly traded REITs.

Similar to publicly traded REITs, nontraded REITs are registered and regulated by the SEC; however, they are not listed and do not trade on an exchange. This makes them less liquid, and potentially more volatile, than publicly traded REITs. Nontraded REITs are sold by broker-dealers, and investors typically pay up to 13%-15% in upfront fees. The tax treatment for both publicly traded and nontraded REITs is similar.

2.7. Scorpio Inc. has a mean return of 19%, and a standard deviation of 25%. What is the probability that the stock will have a return greater than 19% if the returns are normally distributed?

The answer is 50%. In a normally distributed yield curve, half of the returns will be greater than the mean return, and half of the returns will be less than the mean return.

3.11. How can an investor use intrinsic value in deciding what action to take concerning a security?

The intrinsic value of a security is compared to the security's current market price to determine if the security is undervalued (i.e., the market price is less than the intrinsic value) or overvalued (i.e., the market price is greater than the intrinsic value). If an asset is undervalued, the asset may be an attractive investment. If an asset is overvalued, the asset may be less attractive as an investment.

c. market segmentation theory

The market segmentation theory argues that yields are a function of the supply and demand for funds in each segment (short, intermediate, long) of the market.

b. How does a UIT differ from a mutual fund?

Whereas a managed investment company, like a mutual fund, actively manages a pool of assets for its shareholders, a UIT fixes the pool of assets at the outset and is a self-liquidating, passive investment company. Although UITs have front-end sales charges, they do not have annual management fees. Many UITs are organized as grantor trusts instead of investment companies in order to avoid having to meet the diversification tests that are required of investment companies. A UIT has a limited life, whereas a mutual fund has an indefinite life.

4.1. Janice Carlysle owns a ZZT Corporation convertible bond. The bond has a 4.5% coupon rate paid semiannually and matures in eight years. Comparable debt (with the same rating and maturity date) is yielding 6%. Janice's bond is convertible at $25 a share, the current market price of ZZT common stock is $33, and the bond sells for $1,200. a. What is the conversion value of the bond?

$1,320

1.12. Your client asks what the market price of a particular zero coupon bond should be based on current interest rates. The bond will mature in seven years and will pay a face value of $1,000. Comparable bonds (bonds with similar maturities and of the same investment grade) are yielding 4.75%. What should be the price of this bond?

$719.92

3.7; b. The current dividend is $2.20 annually, and it is expected to grow at 5% per year. What is the intrinsic value of the stock using the DDM?

$73.10

2.3. If a taxpayer has a tax liability of $9,139.50 and taxable income of $60,000, what is his average (effective) tax rate?

$9,139.50 ÷ $60,000 = 15.23%

b. What is the investment value of the bond?

$905.79

1.11. Your client asks what the market price of a particular bond should be. The bond pays 3.5% coupon interest semiannually. The bond will mature in seven years and pay a face value of $1,000. Comparable bonds (bonds with similar maturities and of the same investment grade) are yielding 5%. What should the price of this bond be in the secondary market?

$912.32

1.1. The calculation of a bond's price depends on what four factors?

(1) the interest paid by the bond (2) the interest rate available on comparable bonds of the same maturity and grade (market interest rate) (3) the maturity date of the bond (4) the bond's principal or call amount.

1.10. What are the three different types of energy MLPs?

--Upstream MLPs—involved in the exploration, recovery development, and production of crude oil, natural gas, and natural gas liquids. --Midstream MLPs—involved in the gathering, storage, and transportation of oils and gases. --Downstream MLPs—involved in the distribution of the fuels to end users and customers. Products include gasoline, jet fuel, diesel oil, natural gas, and asphalt.

2.5. The Stargazer Fund has a mean (average) return of 10%, and a standard deviation of 15%. Assuming the returns are normally distributed, what range of returns would you expect 68% of the time? ____________________________ 95% of the time? ____________________________ 99% of the time? ____________________________

-5% to +25% (10 - 15 = -5, and 10 + 15 = 25) -20% to +40% (-5 - 15 = -20, and 25 + 15 = 40) -35% to +55% (-20 - 15 = -35 and 40 + 15 = 55)

Capital market line (CML)

-Macro aspect of capital market theory -Specifies the risk/return relationship for a market portfolio that contains all risky assets that exist, with cash equivalents being used to lower the risk of the market portfolio -Standard deviation used a measure of risk

5.4. What are some of the potential pitfalls of mutual fund investing?

-One pitfall to look out for is new funds with great records that were created by putting hot new issues in the fund to generate a superior start-up record in order to impress the publications that rank funds. This can also happen if a fund was an "incubator fund," where the investment company starts a new fund and if it compiles a good record, the company will offer it to the public. -Another pitfall is top-ranked funds that are misclassified, that concentrate in certain sectors to enhance short-term performance, and that are top ranked in one subset of an asset class. -A third pitfall is great fund performances that do not always translate into great performances for all the investors in the fund because of when the investors made their investments in the fund. Related to this is fund performances that do not take into account the sales charges and some of the other fees that may be assessed to the individual investor. -A fourth pitfall includes hidden tax liabilities that may be present when an investor buys a fund just before its capital gain distribution date; a large percentage of short-term capital gains that are taxed at marginal tax rates; and large unrealized capital gains that are due to past performance but that may soon be taxed to an investor who buys into the fund now. -Fifth, watch out for disappearing underachievers, which are funds with poor performances that are merged into good funds or are closed, thereby giving the impression that a fund family has only high-performing funds. -A sixth pitfall is misleading long-term performance, which occurs when most of the excellent returns were earned in the early years and more recent returns are below average.

1.1. What are the three main types of income?

1. Active—includes salary and generally any income that you actively do something to earn 2. Portfolio—includes capital assets, which can generate interest, dividends, royalties, and capital gains 3. Passive—includes businesses that you do not actively participate in (e.g., rental activities)

3.2. What are the six key elements that separate hedge funds from mutual funds?

1. Hedge funds are private investment vehicles for sophisticated investors. 2. Hedge funds tend to be more heavily concentrated and specialized than mutual funds. 3. Hedge funds generally use derivatives to a much larger degree than mutual funds. 4. Hedge funds go both long and short, many mutual funds are long-only. 5. Hedge funds often invest in illiquid and nonpublic securities. 6. Hedge funds often use leverage, and oftentimes the amount of leverage can be substantial.

What are the five core steps in the retirement planning process?

1. Help the clients create a picture of their goals. 2. Create a base plan using average returns. 3. Stress test their plan for return-sequence risk and unsystematic risks. 4. Repeat steps 2 and 3 as required to create the plan that works for them. 5. Use Monte Carlo to compare relative results.

Identify the two rules in setting goals.

1. Make goals specific regarding purpose, amount, and time horizons. 2. Goals must be prioritized.

3.5. Briefly describe the four main hedge fund strategies.

1. Market directional: Includes equity long/short, short-selling, and activist. 2. Corporate restructuring: Includes distressed securities, merger arbitrage, event driven, and Regulation D offerings. 3. Convergence trading: Includes fixed-income arbitrage, convertible bond arbitrage, equity market-neutral, and relative value arbitrage. 4. Opportunistic trading: Includes global macro funds and fund of funds.

1.5. According to the Black-Scholes option valuation model, what five variables affect the price of a call option?

1. The current price of the underlying stock 2. The length of time to the expiration date of the option, expressed in years 3. The variability of the underlying stock price, expressed as the standard deviation of the stock's annual return 4. The risk-free rate of interest, expressed on an annual basis 5. The exercise price (strike price) of the option

What are the seven factors or layers Bengen suggests in building an initial withdrawal rate layer cake?

1. Withdrawal scheme (foundation layer) 2. Asset allocation 3. Success rate 4. Rebalancing interval 5. Super-investor (normally capable of better-than-average returns) 6. Desire to leave a legacy 7. Time horizon

List three key elements of an investment strategy.

1. an identifiable goal 2. a method to attain that goal 3. the competencies and resources to sustain the strategy

List industries expected to do well in the following economic stages... 1. expansion - early stage 2. expansion - middle stage 3. expansion - late stage 4. recession

1. consumer credit, transportation, energy, and consumer cyclicals 2. basic materials 3. capital goods 4. consumer staples and utilities

1.6. Under APT, research suggests that four unanticipated factors have the greatest impact on stock returns. What are these four factors?

1. inflation 2. industrial production (GDP) 3. risk premiums 4. interest rate

What are the three parts of Guyton's definition of a safe initial withdrawal rate?

1. never requiring a reduction in withdrawals from any previous year 2. allowing for systematic increases to offset inflation 3. maintaining the portfolio for at least 40 years

Define three additional attributes of a sound investment policy statement.

1. realistic 2. long term perspective 3. clearly defined

Three factors that undermine investment strategy.

1. unrealistic expectations 2. emotional, undisciplined client 3. inadequate time horizons

3.8. Stock BLQ pays an annual dividend of $2.45; its dividends are expected to increase at 4% annually. The stock has a beta coefficient of .72; the risk-free rate is 2%; and the market return is 10%. The current market price of stock BLQ is $33 per share. a. What should be an investor's required return for stock BLQ?

7.76%

3.7; a. Assume the risk-free rate of return is 1.2%, that the expected rate of return of the market is 7%, and that the stock has a beta coefficient of 1.2. What is the investor's required return for the stock?

8.16%

1.9. Jane Roberts owns a public purpose municipal junk bond that pays 6%. a. Assuming she is in the 35% marginal tax bracket, what yield on corporate bonds would be comparable to the yield on Jane's current investment?

9.23%

3.4. What is a credit default swap (CDS)?

A CDS is like an insurance policy in that it transfers the credit risk between parties. The purchaser of the swap will make payments to the seller of the swap until the maturity date of the contract. CDSs are often sold by banks, insurance companies, and hedge funds. The biggest risk that CDS buyers face is counterparty risk.

1.18. What is the difference between Yankee bonds, Eurodollar bonds, and Eurobonds?

A Yankee bond is a bond issued by a foreign firm in the United States. Yankee bonds are denominated in U.S. dollars. A Eurodollar bond is a bond that is sold outside the United States by a non-U.S. bond syndicate, but is denominated in U.S. dollars. Eurobonds are bonds issued in foreign countries by U.S.-based corporations. The bonds are denominated in dollars, and the U.S. firm makes payments in dollars.

2.6. What is a capital asset?

A capital asset is any asset held for personal use or for investment purposes. The definition does not include assets used in a trade or business.

duty of loyalty

A fiduciary must be loyal to his or her clients and must always look out first and foremost for what is in their best interest. The duty of loyalty requires that the client's interests be put ahead of one's own and that all actions be made solely for the benefit of the client.

3.1. What is a fund of funds (FOF)?

A fund of funds (FOF) is a hedge fund that consists of several, usually 10-30, hedge funds. The point is to achieve diversification, but the extra layer of management means an extra layer of fees.

2.1. What is a futures contract? What are the three main types of futures contracts?

A futures contract is a commitment to deliver or receive a designated amount of a specified commodity, financial asset, or currency during a specified month in the future. The seller of the contract agrees to make the specified future delivery, and the buyer agrees to accept and pay for it during the designated month. The three types of futures contracts are commodity futures, financial futures, and currency futures.

1.1. Explain how portfolio risk (σp) is used to develop the efficient frontier in the Markowitz model.

A graph is constructed in which the horizontal axis measures the risk associated with the portfolio (σp) and the vertical axis measures the expected return of the portfolio. Various combinations (portfolios) of securities are plotted on the graph, and a curve emerges that defines the efficient set (frontier) of portfolios, which are those portfolios that offer the highest return for any given level of risk. No single portfolio on the efficient frontier is superior to any other single portfolio on the frontier. Each portfolio has a different risk level and a return consistent with that risk level.

3.12. Robert Berens, age 65, is retiring and has $150,000 to invest. He is interested in purchasing fixed-income securities to provide for his income needs during retirement. Robert will not have any other substantial income, and he will be in the 12% marginal income tax bracket. He has invested in bonds in the past, and he plans to be actively involved in this investment. What kind of fixed-income security is appropriate for Robert, and why? (Consider type, risk rating, marginal tax bracket, term, and other relevant factors.)

A high-grade corporate bond (AA or AAA), a Treasury note or bond, or a federal agency security like a Ginnie Mae would be appropriate. All of these can be bought at par, pay periodic income, and have a high degree of marketability. Because he is in a low marginal tax bracket, taxable securities would most likely provide more after-tax income than municipal bonds. An intermediate term of seven to 15 years would give adequate yield with only moderate interest rate risk.

3.4. What are characteristics of managed accounts?

A managed account, commonly called a separate account or a privately managed account, is one that is managed by a professional money manager who creates a portfolio tailored and customized to the needs of an individual. Usually these portfolios concentrate in a specific sector of the market such as large-cap stocks, small-cap stocks, or a narrower segment like mid-cap value stocks. Managed accounts may have a minimum investment requirement as low as $50,000, but some accounts have minimum investment requirements of $100,000 or $250,000. Fees vary but typically are about 1%-2% of assets under management.

2.4. What is a market anomaly?

A market anomaly is a strategy or situation that cannot be explained away and that would not be expected to occur if the efficient market hypothesis were completely true.

1.7. What is a mutual fund shareholder's tax basis?

A mutual fund shareholder's tax basis is usually the purchase price of the shares being redeemed, including any commissions paid on the purchase, and any reinvested dividends. Basis is important because this amount is subtracted from the redemption proceeds to determine the gain or loss for tax purposes.

5.2. What is a principal-protected product?

A principal-protected product is a type of structured product that offers the return of principal and possibly additional payments contingent on the value and performance of underlying reference assets. A principal-protected product typically has two components: a debt instrument component and a derivative component. The final payoff amount will be determined by the participation rate and terms, the minimum or maximum return set by the issuer, the initial valuation date used, and the final valuation date. The two main ways to calculate a return are using either an average over time, or a point to point calculation. Both approaches are subject to caps and floors that are set by the issuer.

c. redemption fees

A redemption fee is a fee of up to 2% for redeeming shares within a certain time of purchase, such as 90 days or one year. These fees are designed to discourage investors from selling within a short time and buying another fund.

Explain the taxation of municipal bonds.

Almost all municipal bonds are exempt from federal income tax. They are generally free from state tax in the state of issue. Private activity bonds issued after August 15, 1986 continue to generate income that is tax exempt; however, the interest is included in the alternative minimum tax computation. For private-activity municipal bonds issued in 2009 and 2010, the interest is not an AMT preference item. For tax-exempt bonds purchased after April 30, 1993, the amount of market discount recovered upon sale is taxed as ordinary income. For tax-exempt bonds purchased at a premium, the premium must be amortized but the premium is not deductible. The bond owner must reduce his or her basis each year by the amount of premium allocable to the year.

2.1. What is an efficient market?

An efficient market is one in which prices fully reflect all known information quickly and accurately. In an efficient market, investors are competitive, information is widely available, new information is generated randomly, and investors react quickly to the new information. A security's equilibrium price is a true valuation of what the market believes the security is worth; thus, an investor cannot expect to consistently beat the market over time. What the investor can expect to earn is a return that is consistent with the market return and the amount of assumed risk. A security can never be overvalued or undervalued, so analysis undertaken to discover mispricing is useless.

5.3. What variables enter into calculating the after-tax return earned on an investment in a mutual fund?

As with other investments, calculating the after-tax return on a mutual fund investment is based on (1) the individual's marginal tax rate, (2) the increase in the fund's net asset value, (3) the fund's distributions, and (4) whether the individual sells shares to pay taxes or pays them out of pocket.

3.3. What are characteristics of business development companies (BDCs)?

BDCs invest primarily in privately owned small to mid-size companies. Oftentimes bonds are purchased by a BDC, and the yield on these bonds will be high since the companies involved are typically smaller and are not well-established, meaning their cost of credit is higher since their credit risk is higher. There can be warrants attached to the bonds, which would provide additional upside potential for the BDC investor. BDCs must distribute at least 90% of their investment income, and they are structured as a flow-through entity, meaning any dividends are taxed as ordinary income and are not eligible to be taxed as qualified dividends. Primary risks to be aware of when investing in BDCs include credit, leverage, interest rate, market, and liquidity risk.

3.13. John Bloom, age 49, wants to take early retirement next year when he turns 50. He wants to invest $200,000 in a fixed-income security to provide him with additional income. He estimates that he will be in the 32% marginal tax bracket. He has invested previously, and he is willing to be aggressive with this investment to increase his return. What kind of fixed-income security is appropriate for John, and why? (Consider type, risk rating, marginal tax bracket, term, and other relevant factors.)

Because of his high tax bracket, municipal revenue bonds are appropriate, assuming their equivalent yield exceeds the yield of corporate bonds. Purchasing bonds with lower ratings (BB or BBB) would be consistent with his aggressive attitude of attempting to increase his return while realizing additional income from this investment. If rates fall, longer maturities may be appropriate to provide capital gain potential.

What is the significance of the correlation coefficient to the accurate interpretation of the meaning of beta?

Beta is significant if properly calculated. Intuitively, an investor might conclude that a stock with a standard deviation that is twice the standard deviation of the market would have a beta of 2.0. The computations show that to be the case only when the stock in question is highly correlated with the market. If a stock has a low positive correlation (+0.25), the beta is only 0.5. Many investors will conclude that a stock with a beta of 0.5 is half as variable as the market. What this computation should actually tell them is that they cannot use beta to judge the volatility of the stock, but instead must refer to the stock's standard deviation to measure the volatility of the stock. Beta cannot be accepted blindly. Investors must know the stock's correlation with the market index, or benchmark, against which its beta is calculated.

3.3. How are collateralized debt obligations (CDOs) structured?

CDOs are structured financial products that repackage a pool of cash flow-generating assets into tranches that can be sold to investors. Interest and principal payments are made in order of seniority so that senior tranches have the least risk. Junior tranches will have higher default rate and thus higher yields, and senior tranches will have lower yields but higher credit ratings.

3.1. What are collateralized mortgage obligations (CMOs)?

CMOs are a type of complex debt security that repackage and direct the payments of principal and interest from a collateral pool of mortgages to different credit tranches based on their risk profiles.

2.7. Explain the taxation of capital gains and losses.

Capital gains and losses are divided into short term and long term. Long-term capital gains (LTCG) and long-term capital losses (LTCL) are netted, as are short-term capital gains (STCG) and short-term capital losses (STCL). If a gain and a loss remain, these are again netted. Net LTCG are subject to a 0% rate if the taxpayer's taxable income is less than $40,000 for single filers or less than $80,000 for those who are married filing jointly (MFJ). Net LTCG are subject to a maximum tax rate of 15% if the taxpayer's taxable income is between $40,000 and $441,450 for single filers or between $80,000 and $496,600 for those who are married filing jointly (MFJ). The LTCG rate is 20% for taxable income over $441,450 for single filers or over $496,600 for those who are married filing jointly (MFJ). A maximum 28% rate applies to net gains on collectibles, if held for more than one year. A maximum 25% rate applies to unrecaptured Section 1250 income-the gain created by straight-line depreciation on realty. An STCG is treated as ordinary income subject to a marginal tax up to 37%. A net capital loss is deductible only up to $3,000 per year, with a carryforward of the remainder.

4.4. What are some of the cost and benefits of investing in leveraged and inverse mutual funds and ETFs?

Costs include management fees, interest expenses, transaction costs (buying and selling derivatives), as well as trading costs (commissions) and taxes for investors. Leveraged and inverse ETFs do not have the same kind of tax efficiency as traditional ETFs, which can lead to significant capital gain payments for shareholders. Leveraged and inverse ETF benefits include offering an easy and relatively inexpensive way to use leverage without using options or margin. They work well for short-term and active traders who have a strong opinion about the short-term direction of the market, and they can be easily traded intraday whenever the stock exchange is open.

1.2. What is the role of credit rating agencies, and what effect do their ratings have on the marketing of debt securities and on the interest rate paid by the issuer?

Credit rating agencies, such as Moody's and Standard & Poor's, have credit rating systems that indicate the likelihood of default by issuers of debt (e.g., corporations and municipal entities). If the rating of a bond issued by a firm or municipality is low, or if it declines from the original rating, financial institutions and individual investors may not be inclined to purchase the bond. In fact, many financial institutions are prohibited by law from purchasing debt with a credit rating below investment grade. The coupon interest rate required by investors increases as an issuer's credit rating decreases.

ERISA of 1974

ERISA was enacted to stem company retirement plan abuses and to make sure that employees were protected and that they would be paid any promised benefits. ERISA set standards for participation, vesting, funding, reporting, and disclosure. This act also established the Pension Benefit Guaranty Corporation (PBGC). ERISA requires that anyone giving investment advice to a company retirement plan must be a fiduciary.

3.1. What is an exchange-traded note (ETN)?

Exchange-traded notes are debt securities usually issued by large commercial banks or investment banks that, like ETFs, track an index and trade like stock. As a result, from the investor's perspective, ETNs look and seem like ETFs. Unlike ETFs, ETNs do not hold a basket of stocks but rather represent promises by the issuers to match the returns of an index or commodity, less fees. As such, investors need to be concerned about the creditworthiness of the issuer; because, in the event of default, investors would wait in line for payment with other creditors as there are no securities to back the ETNs.

c. When estimating the dividend growth rate, what impact do fluctuating earnings have on the computation?

Fluctuating earnings could result in a dividend growth rate that is much lower or higher than is realistic, especially if one or two recent years are not typical. To counter this tendency, analysts could use a longer time period and/or eliminate the atypical years.

3.2. What are characteristics of guaranteed investment contracts (GICs)?

Guaranteed investment contracts are issued by insurance companies to pay either a fixed rate (called nonparticipating) or, less commonly, a variable rate (called participating) depending on interest rate fluctuations. Monies received by the insurance company are invested in bonds and sometimes stocks that are held in their general account. As such, contract holders are creditors of the insurance company in the case of insolvency—an unlikely, but not impossible, situation. These contracts are usually issued for one to five years and then renewed. They do not fluctuate in price, are generally regarded as safe investments, and therefore are similar to certificates of deposit except they are not guaranteed or insured by the U.S. government; instead, the guarantee is from the insurance company, so the creditworthiness of the company is an important factor.

4.12. How should historical returns be used in investment planning?

Historical returns may help determine what rates of return are reasonable for use in projecting future returns. Straight-line projections of historical returns into the future can result in unrealistic expectations about future returns. Frequently, asset classes that outperform the historical average in one time period underperform the historical average in the subsequent time period.

5.7. What bond portfolio actions should investors take if they expect interest rates to increase? What such actions should investors take if they expect interest rates to decrease?

If interest rates are expected to increase, the yield curve will shift upward, causing capital losses in bond portfolios. An investor who wants to minimize capital losses should move from long-term bonds to short-term bonds. This action would also allow the investor to reinvest at increasingly higher yields as rates increase. Here, reinvestment risk would work in the investor's favor. If the investor expects rates to decrease, the yield curve would shift downward, causing capital gains in bond portfolios. In this case, an investor should move from short-term bonds to long-term bonds in order to lock in higher coupons and to generate capital gains in the bond portfolio.

Assume that an individual would like to leave a bequest (at death) of $500,000 to a charity. The individual is 65 years old, expects to live for 25 more years, and can earn 7% on invested assets. How much will the individual need to deposit today in order to give the $500,000 to charity 25 years from today?

If the individual deposits $92,124.59, and earns 7%, this sum will grow to $500,000 in 25 years. Keystrokes: 25 N, 7, I/YR, $500,000, FV, Solve for PV = $92,124.59 If the individual wanted to give the inflated value of $500,000 (at 3.5% inflation over 25 years), the starting point would be $1,181,623. Discounting this amount back to today (at 7%), the deposit would have to be $217,712.97.

e. What is the downside risk percentage of the bond?

If the price of the underlying stock falls substantially, the maximum that the price of the bond can fall is about 24%. You always use the difference between the market value and the investment value (not conversion value, even if higher) to determine downside risk.

1.19. Characterize income-mixed funds in terms of the investment objectives of the fund and types of securities held by the fund.

Income-mixed funds have a primary objective of current income and a secondary objective of growth. They invest primarily in dividend-generating common stock, but sometimes they also own preferred stock, high-grade bonds, and convertible securities. The current yield is less than that of comparable bond funds but more than that of growth stock funds. These funds have slightly more volatility than bond funds and slightly less volatility than growth funds.

1.26. In addition to domestic equities, what other asset classes should investors consider when assembling a fully diversified portfolio of mutual funds?

International equities should also be included, which can include stocks from countries in developed, developing (emerging markets), and even frontier markets. Some international stock funds concentrate in small-cap stocks. Fixed-income funds also diversify a portfolio and can contain either U.S. or international bonds. U.S. bond funds offer income that is taxable at the federal level (U.S. government and corporate bonds) or tax-exempt municipal bonds. Investors in a relatively high marginal tax bracket generally favor tax-exempt bond funds. International bond funds offer additional diversification. Commodity funds are increasingly used as another asset category to be included in a portfolio. Owning funds in from four to seven different asset classes generally is considered sufficient to have a fully diversified portfolio.

1.1. How does an investment company serve as a tax conduit?

Investment companies act as conduits by distributing dividends, interest, and realized capital gains to their shareholders who pay taxes on distributions. The funds pay no federal tax on the income they earn. To qualify as a regulated investment company, a fund must earn at least 90% of all income from security transactions and distribute at least 90% of all interest and dividend income. In addition, minimum diversification requirements must be met.

loss aversion bias

Investors do not like to lose, and there is a tendency to keep losing investments (in the hopes of breaking even) and selling winners, which in the long run will create a portfolio of losers. Some stocks may never get back to the investor's purchase price, or may take a long time to recover. In the meantime, the money could have been invested more productively elsewhere.

2.14. What are some of the benefits and risks of investing in managed futures?

Managed futures offer diversification benefits for long-term investors because they typically have a low correlation with traditional stocks and bonds. They can provide a positive return when stocks and bonds are negative. There is, however, lack of transparency about how any given managed futures fund is operated, and high fees can offset what would have otherwise been a successful and profitable strategy.

4.17. Should most portfolios be measured against just one benchmark? Why or why not?

Most portfolios should be measured against a blended—and not just one—benchmark. Unless a portfolio is composed of just one asset class (such as entirely in U.S. large-cap stocks) then comparing a portfolio comprised of various asset classes against a benchmark representing just one class does not make sense. The percentages should be weighted so that the benchmark percentage used matches the percentage found in the client's portfolio. For example, if a client's portfolio is composed of 25% in U.S. large-cap stocks, then only 25% of the benchmark being constructed for comparison purposes should be represented by the S&P 500.

1.18. Characterize multisector bond funds in terms of the investment objectives of the fund and types of securities held by the fund.

Multisector, or strategic income, bond funds buy U.S. government bonds, corporate high-yield bonds, and foreign bonds. Their objective is high current income with lower volatility than that found in bond funds owning only one type of bond. Because these types of bonds are affected by different economic forces, each bond's price volatility should tend to offset the others, thereby, leading to less volatility overall.

a. national funds

National funds provide geographic diversification. These funds hold municipal bonds from many states; therefore, most of the income from these funds generally is subject to state income taxation. These funds typically invest in intermediate- and long-term bonds, thereby providing moderate to high returns at moderate to high risk levels.

c. What advantages and disadvantages are associated with UITs?

On the positive side, unit investment trusts provide investors with fixed income, diversification, ease of ownership, highly predictable income streams, and high-quality securities. Some trusts include insured bonds. Because UITs are not managed, they have no management fees, and the annual fees are low. On the negative side, although the sponsoring organization usually will maintain a secondary market in trust shares, investors may have to sell their interests at a loss if inflation or interest rates increase. There is always the chance that an issuer of the debt held by the UIT will have financial problems and/or default. UITs consisting of stocks offer certainty of time but not certainty of price. At the date the UIT terminates, the stocks could be selling at historically low prices due to a recession, high inflation, or some other economic or market factor. Stock UITs do not have the flexibility that an investor would require of an equity portfolio.

3.3. Why is present value analysis used to calculate intrinsic value under the dividend growth model?

Present value analysis is used because the value of any security can be determined by discounting the future stream of economic benefits (cash flows—generally dividends) that the investor expects to receive.

2.1. What factors determine the amount of price fluctuation in a bond?

Price fluctuations are affected by a bond's grade (credit/default risk), its coupon rate, its time to maturity, its duration, and any changes in market interest rates.

3.2. How do private REITs differ from nontraded REITs?

Private REITs, just like nontraded REITs, are not listed or traded on an exchange. Private REITs differ from nontraded REITs in that they are not registered with the SEC, and should only be sold to accredited investors.

1.3. Can the Markowitz model by itself be used to select an optimal portfolio for an investor? Explain your answer.

Rational investors want an optimal portfolio that offers the highest return for a given level of risk or the least amount of risk for a given level of return. By itself, the Markowitz model does not identify the optimal portfolio; it only identifies the efficient frontier of portfolios. The optimal portfolio occurs at the point of tangency of the indifference curve and the efficient frontier. The optimal portfolio for an investor is determined by combining the efficient frontier and the investor's willingness to bear risk.

2.6. List several forms of real estate ownership and identify the main ways in which they differ.

Real estate ownership can occur in the following forms: outright ownership, general partnership (joint venture), limited partnership, corporation, and REITs. These forms of ownership differ in terms of the liability incurred by the investor, the management efforts required, the income tax implications, the ease with which the investors' ownership interests are transferred, and the continuity of the entity upon the death of the various participants in the investment.

What is a reverse mortgage?

Reverse mortgages are a special type of home loan where the payment stream is reversed (that is, the lender pays the homeowner a stream of income secured by the considerable amount of equity in the home). The lender makes payments to the homeowner on the basis of the fair market value of the home and the age of the borrower at the time that the loan is made.

4.5. What are some of the risks associated with investing in leveraged and inverse mutual funds and ETFs?

Risks include compounding risk, leverage risk, lack of diversification risk, liquidity risk, stock market risk, valuation risk, short sale exposure risk, and portfolio turnover risk. There are also numerous risks when using derivatives, including correlation risk and counter-party risk. Special risks associated with ETFs include intraday price performance risk (due to the need to rebalance each day), longer term price performance risk, trading issues (such as trading being halted), and stock market variance risk. These are complex investments, and both the adviser and the client should understand the risks involved.

Mary has determined that she needs income of $5,000 per month for 25 years of retirement. She has determined that, with a 7% return, but not reflecting inflation, she will need a little more than $700,000 to fund her retirement. If she factors in a 3.5% rate of inflation, what will happen to the amount needed to fund her retirement?

She will need slightly more than $1 million to maintain an inflation-adjusted budget with equal purchasing power. Set calculator for 12 P/YR Keystrokes: 25, DOWNSHIFT, N (300 compounding periods), 3.3816, I/YR (inflation-adjusted return), 5,000, PMT, Solve for PVAD = $1,014,390

3.6. What is smart beta?

Smart beta is a combination of both indexing and active management, whereby the fund manager attempts to overweight assets in the benchmark that he thinks will outperform, and underweight assets that he believes will underperform.

d. 12b-1 fees

Some funds impose 12b-1 fees, which are used to pay for the costs of marketing and distributing fund shares or as, in effect, a trailing commission to the seller of the fund. A 12b-1 fee is a continuous annual fee, unlike a front-end load, which is charged only when shares are purchased. Under FINRA rules, the maximum portion of the 12b-1 fee that may be devoted to distribution costs is 0.75% of assets. This fee is used to recover the sales commission. A 0.25% "service fee" many also be imposed to cover annual trailing commissions to the salesperson, for a total 12b-1 fee of up to 1.0%, which is included in the fund's expense ratio.

standard deduction

Standard deduction refers to an allowance granted to most individuals in lieu of itemizing deductions. A taxpayer has the option of using the greater of total itemized deductions or the standard deduction.

b. state funds

State funds have the objective of providing tax-free income, at both the federal and state levels, for the residents of a particular state, and they do so by investing in bonds issued only within that state. For in-state residents, the returns of single-state funds are generally higher than are those of diversified intermediate- or long-term funds. The risks depend on the average maturities and the credit ratings of the bonds in the portfolio.

The Andersons are a 50-year-old couple with an annual retirement budget of $95,000. They want to plan for a retirement life expectancy of 25 years (starting at age 65), and assume a 3.5% average inflation rate and a 7% long-term rate of return. How much money will they need at age 65 to fund their retirement?

Step 1: Find the inflated value of $95,000 in 15 years. 15, N; 3.5 I/YR, 95,000, PV; solve for FV = $159,158 (rounded). So, the equivalent value of $95,000, inflated for 15 years at 3.5%, is $159,158. This becomes our starting income payment for the second step. Step 2: Calculate the PV of 25 years of payments, using a first payment amount of $159,158, and factoring both inflation (3.5%) and the rate of return (7%) (i.e., a serial payment). This requires two steps. First, determine the inflation-adjusted interest rate, then calculate the serial payment PV. To determine the inflation-adjusted interest rate, do the following: ([1.07 ÷ 1.035] - 1) × 100, becomes ([1.0338] - 1) × 100 = 0.0338 × 100 = 3.3816 So the inflation-adjusted interest rate is 3.3816. This is what we will use to calculate the PV for the sum required at the beginning of retirement. Make sure you are in the BEG mode. 25 N; 3.3816 I/YR, 159,158, PMT; Solve for PVAD = $2,747,052 (Note: Your answer may vary slightly depending on the exact interest rate entered). This calculation lets us know that the Andersons will need over $2.7 million in 15 years to fully fund their retirement budget.

In what one way does the Sharpe ratio differ from the Treynor ratio?

The Sharpe ratio uses the standard deviation of the portfolio rather than its beta in the denominator of the equation.

the "adventurer"

The adventurer is confident, but impetuous or intuitive in his or her approach to decision making; typical clients in this category are entrepreneurs.

2.8. Libra Inc. has a mean return of 11%, and a standard deviation of 9%. Assuming the returns are normally distributed, what is the probability that the stock will have a return greater than 20%?

The answer is 16%. We know that half of the returns are going to be greater than the mean return of 11%. We also know that one standard deviation (which will be evenly distributed) accounts for 68% of the returns and would range from +2% to +20%. Half of the 68% of returns would be above the mean return, and half below. So 34% of the returns would be greater than the 11% mean return, and would fall between 11% and 20%. If we know 50% of the returns will be greater than 11%, and we know that 34% of the returns will fall between 11% and 20%, we then know that 16% (what remains of the 50%, 50 - 34) of the returns will be greater than 20%.

3.8. If you want to ensure that $40,000 is available in 13 years when your child is about to enter college, would you select a zero coupon bond that matures in 13 years or a coupon bond that matures in 13 years?

The appropriate bond is the one with duration close to the duration of the goal. The goal's duration is 13 years. The appropriate bond is the zero coupon bond, because a zero's duration is equal to its maturity. The duration of a coupon bond is less than its maturity. The coupon bond's duration must be less than 13 years, because its maturity is 13 years.

1.3. Other than the sources of risk identified in the previous question, what disadvantages are associated with investments in tangible assets?

The bid-ask differential is much greater with tangible assets than with financial assets. In addition, the commissions on some tangible assets are a large percentage of the transaction price. Tangible assets produce little or no income, and they must be insured to protect them from loss from fire, theft, and other perils. Investors also face the possibility of price markups that are well above the asset's intrinsic value.

the "celebrity"

The celebrity is a fashion follower who wants to be "where the action is" but who lacks the confidence to get there without guidance.

d. What is the bond's conversion premium?

The conversion premium is $120, the difference between the bond's market price of $1,200 and the bond's conversion value of $1,320.

a. If a company decides to raise its dividend payout ratio (and its future return on equity [ROE] is projected to remain constant), how would the decision to raise the dividend payout ratio affect its dividend growth rate?

The dividend growth rate will be lower because the company will retain less of its earnings to finance its future growth. Should the company need additional funds for future expansion, it will have to seek those funds through a stock or debt offering because the shareholders will have already received a portion of the money that could have been used to internally finance growth.

duty of care

The duty of care requires the fiduciary to have the competency to give fiduciary advice, which requires a certain level of knowledge and skill. Good intentions alone are not enough. Fiduciaries have to be competent enough to give advice, and if they delegate or consult with others, they need to be competent enough to vet other experts.

duty to consult

The duty to consult requires consulting with experts when the investment professional might not have the knowledge and experience needed in a particular situation. With the increased specialization and complexity in the field of investments, no one individual has the expertise to be fully competent in all areas. Where that expertise is missing, there is the duty to consult with someone who has that expertise.

duty to diagnose

The duty to diagnose covers the obligations to "know your customer" and to investigate the suitability of any products recommended as investments. This ethical duty is supported by formal requirements. FINRA Rule 2111 stresses the importance of learning all the essential facts about a client and that client's account. The FINRA suitability rule requires a broker-dealer to have reasonable grounds for believing that a recommended investment is suitable for the particular client—suitable in terms of the client's other security holdings, financial situation, investment goals, risk tolerance level, tax situation, and other individual circumstances.

duty to disclose

The duty to disclose involves disclosing all material facts and all conflicts of interest as required by government regulations and professional codes of conduct. Full disclosure on new issues is a requirement of the Securities Act of 1933. The investment professional also is ethically bound to disclose any negative or risk factors involving investments being offered to clients.

3.4. Assuming the correlation of a stock and the market is high, what would beta coefficients of 0.5, 1.0, and 1.5 mean to someone investing in the stock?

The larger the beta coefficient, the more volatile the security's historic price is relative to the market. Investors often use beta as a rule of thumb for estimating the percentage change in a stock's price when the overall market moves X%. A beta of 1.0 indicates the movement of the stock would be expected to be identical to the movement of the market. A beta of 0.5 means that the price of the stock has been less volatile than the market; for example, if the overall market changes by 4%, the stock would be expected to change by 2%. If a stock has a beta of 1.5, the price on the stock would be expected to change by 6% when the market changes by 4%. Stocks with high beta coefficients are referred to as "aggressive," while stocks with low beta coefficients are referred to as "defensive."

Explain the taxation of mortgage-backed securities.

The monthly payments from mortgaged-backed securities consist of part interest and part principal. The interest portion is taxable as ordinary income; the return of principal is not taxable but serves to reduce the holder's basis in the security.

a. How is NAV calculated?

The net asset value of an investment company is the total asset market value minus any liabilities. The net asset value per share is the NAV divided by the number of shares outstanding.

Using a 3.5% inflation rate, what will happen to the purchasing power of $5,000 over a 25-year period?

The purchasing power will have decreased to less than half of what it was at the beginning of the period ($2,116 instead of $5,000). In other words, in 25 years, almost $12,000 will be required to have the same purchasing power as $5,000 (at a 3.5% inflation rate).

Briefly discuss the net investment income tax (NIIT).

The tax is 3.8% of the lesser of: --the net investment income --the excess of the modified AGI (MAGI) over a specified threshold amount -The threshold amount is $250,000 for married taxpayers filing jointly and $200,000 for single or head of household. -Investment income includes the income from interest, substitute interest, dividends, substitute dividends, passive business income, annuities, net capital gains, net royalties, and net rental income. -Certain income specifically is not included in investment income—income from active trades or businesses, income from qualified plans—including 401(k)s, 403(b)s, and IRAs, Roth IRAs, and 457 plans. -Net investment income is investment income reduced by deductible investment expenses.

a. weak form

The weak form of the EMH suggests that historical price data and other technical indicators of the market are of no value in predicting future price changes (i.e., technical analysis will not produce superior results). Fundamental analysis, however, may produce superior investment results.

5.4. What are the basic characteristics of steepeners and range accrual notes (RANs)?

These are both examples of complex, interest rate structured linked products that are only suitable for sophisticated investors with certain expectations about the direction of interest rates. Steepeners are a type of interest rate swap where one party agrees to pay the other a fixed rate in exchange for a floating rate. Steepeners perform best during periods of rising interest rates. Range accrual notes (RANs) are linked to a particular reference index, such as LIBOR, and consist of embedded options giving the issuer the right to any income outside of an "accrual range." For example, a RAN tied to the six-month LIBOR might have a range of 0.35%-0.65%. As long as the LIBOR stays within this range, the investor will receive income. If the RAN goes outside of this range (on either the upside or downside), then no interest will be paid to the investor.

itemized deductions

These are deductions for expenses that are generally personal in nature. Examples include deductions for property taxes and mortgage interest on a residence, medical expenses exceeding 7.5% of AGI (2020), charitable contributions and investment interest expenses. These often are referred to as "below-the-line deductions" or "deductions from AGI."

Explain the taxation of dividends received from a life insurance policy.

These dividends generally are nontaxable because they are a return of capital. However, they are taxable in three instances: 1. They are taxable as ordinary income to the extent that the dividends received exceed the investment in the contract. 2. They accumulate and interest on the accumulated dividends is paid. The interest is taxable as ordinary income. 3. With a modified endowment contract, dividends received as cash or used to pay a loan are treated as a distribution, which is potentially taxable. Dividends retained by the insurer to pay premiums or to purchase paid-up insurance do not create a taxable event.

Securities Act of 1933

This act applies to most new, publicly issued securities. The purpose is to require the "registration" of securities with the Securities and Exchange Commission (SEC) by providing full disclosure in the registration statement of the securities that the issuer or vendor was about to issue. If the SEC finds misleading, incomplete, or inaccurate information, it will delay the offering until the registration statement is corrected. All new issues must be accompanied by a prospectus.

Sarbanes-Oxley Act of 2002

This act set up the Public Company Accounting Oversight Board, which consists of five financially literate members, two of whom must be or have been certified public accountants. The board is to establish, or adopt, by rule, "auditing, quality control, ethics, independence, and other standards relating to the preparation of audit reports for issuers." It also must conduct inspections of accounting firms, conduct investigations and disciplinary proceedings, and impose appropriate sanctions. It requires a company's chief executive officer and chief financial officer each to certify the financial and other information contained in the issuer's quarterly and annual reports. The rules also require these officers to certify that they are responsible for establishing, maintaining, and regularly evaluating the effectiveness of the issuer's quarterly and annual reports about their evaluation and whether there have been significant changes in the issuer's internal controls or in other factors that could significantly affect internal controls subsequent to the evaluation. This law also makes it generally unlawful to extend credit to any director or executive officer. This act has precipitated the employment of chief compliance officers.

Securities Exchange Act of 1934

This act, which established the Securities and Exchange Commission and gave it enforcement powers, aimed to regulate securities transactions on both organized exchanges and in over-the-counter markets. This act forbids market manipulation, deception, misrepresentation of facts, and fraudulent practices, and requires most broker-dealers and transfer agents, clearing agencies, and self-regulatory organizations (including securities exchanges) to register with the SEC. In this way, the SEC supervises and regulates many aspects of exchanges. This act also requires many issuers of securities to provide ongoing information about their business affairs by filing quarterly financial statements with the SEC, sending annual reports to shareholders, and filing 10-K reports (which have more financial information than the annual reports), with the SEC annually. Further, it made subject to law the trading activities of corporate directors and officers ("insiders"), who were viewed as principal villains in the market debacle of 1929. In the last two decades, the SEC has widened the interpretation of an insider to include anyone having information that was not public knowledge. This act also gave the Federal Reserve Board of Governors responsibility for setting margin requirements when buying securities.

c. When the economy appears to be close to entering a recession, how will the yield curve change, and why?

To prevent a serious recession, the Fed will probably decrease the Fed funds and discount rates so that short-term rates will fall rapidly, which will turn the yield curve from a negatively sloped or relatively flat curve to a positively sloped curve. Long-term rates will fall because the market will know that the Fed is increasing liquidity and that inflation expectations are probably lower. Long-term bonds will experience capital gains in addition to coupon yield, giving investors excellent total returns.

1.11. Why is the concept of total return more important than current yield when considering bond funds?

Total return, important for all funds but especially important for bond funds, provides the best overall measure of investment performance because it incorporates current yield plus any change in the securities' values. Novice investors often consider only current yield without regard to the fluctuation in a fund's net asset value.

1.11. Explain the characteristics of Treasury STRIPS.

Treasury STRIPS are created by government bond dealers through the separation of principal and interest payments, called coupon stripping. STRIPS are derived from Treasury notes and bonds; however, individual investors do not buy them directly from the Treasury. Instead, STRIPS may be purchased through financial institutions and government securities dealers. The payment of interest and principal on the note is a direct government obligation, there is no risk of default.

What limitations are associated with the Treynor ratio?

Treynor does not indicate whether a given portfolio manager outperformed or underperformed the market. Also, as with Jensen, Treynor assumes that the portfolio is part of a fully diversified portfolio.

2.2. How are asset-backed securities (ABSs) created?

Typically, a special purpose vehicle (SPV) is created to handle the securitization of asset-backed securities. The SPV bundles the pool of assets into different tranches that each have different risk/return profiles.

4.5. Is one of the three performance measurements (Jensen, Sharpe, or Treynor) better than the others?

Whether one performance measure is preferred over another by an investor depends on the investments being evaluated. The Sharpe ratio encompasses total risk, and it would seem to be more appropriate if an investor's total portfolio is not well diversified. If an investor is concerned with evaluating the performance of a mutual fund, for example, and the fund represents the investor's total risk, the Sharpe ratio would be preferred. If the goal is to evaluate how the portfolio performed relative to the market (alpha) and/or to other portfolios, Jensen would be used. If the investor's entire portfolio is well diversified, then the Treynor ratio would be appropriate.

1.1. How do the markets for tangible (or physical) assets differ from the markets for financial assets?

While stocks and bonds are traded on organized security markets and over-the-counter markets, physical assets are bought and sold on informal markets in which price quotations are not readily available. Unlike the markets for financial assets, these informal markets do not have information regarding transaction volume readily available, they have little or no regulation, and they can sometimes be inefficient.

2.8. Explain an initial margin in a commodity futures transaction. Describe how the initial margin differs from the margin that is used in purchasing stocks or bonds.

With a futures contract, there is no borrowing required on the part of the investor in order to finance the balance of the contract. The initial margin, or deposit, is the minimum "good faith" amount paid by the investor to ensure completion of the contract and represents collateral that would cover any loss in the market value of the contract that might arise from adverse price movements. An initial margin does not vary with the dollar amount of the transaction, as a margin for stock does. Each exchange sets its own minimum initial margin requirements for each contract in dollars.

duty to keep current

With all the changes inside and outside the securities industry, all those operating in the industry have an ethical obligation to keep current with developments that affect their clients. These developments can be tax law changes, changes in the economy, new product development, or changes in the fortunes of individual securities issuers.

b. yield-to-maturity

YTM is the compound yield earned on a bond from the time it is purchased until its maturity date. It includes both the periodic cash income received and any capital gains or losses that arise because the principal amount is greater or smaller than the current market price. YTM is the market rate of return, the interest rate that equates the stream of interest payments and the par value at maturity to the bond's current price.

b. What sort of characteristics would you look for in a bond chosen for a client with a moderate risk tolerance?

a bond with a moderate duration (five to 10 years) and an intermediate maturity (seven to 15 years), a coupon that is near current market rates, and a low investment grade credit rating, such as A or BBB

1.8. Explain the characteristics of Series EE and Series I bonds according to the following categories. a. maturites b. minimum purchase c. tax treatment d. method of purchase e. accrual period f. interest rate

a. maturities Series EE Bonds: 30 years Series I Bonds: 30 years b. minimum purchase Series EE Bonds: $25 Series I Bonds: $25 c. tax treatment Series EE Bonds: Federal tax can be deferred; they are exempt from state tax. Series I Bonds: Federal tax can be deferred; they are exempt from state tax. d. method of purchase Series EE Bonds: electronically through TreasuryDirect, and employer payroll deduction plans Series I Bonds: electronically through TreasuryDirect, and employer payroll deduction e. accrual period Series EE Bonds: monthly Series I Bonds: monthly f. interest rate Series EE Bonds: fixed return Series I Bonds: fixed return plus an inflation premium reset semiannually

1.11. For each of the following types of asset allocation decisions made, which kind of asset allocation strategy is being employed? a. the decision to reallocate 40% of fixed income to intermediate-term bonds and 60% to short-term bonds b. the decision to maintain an allocation of 60% stocks and 40% fixed income c. the decision to reallocate 30% of the large-cap asset class to value and 70% to growth d. the decision to allocate 75% to stock and bond index funds and 25% to investments adding diversification and/or reducing risk

a. tactical allocation b. strategic allocation c. tactical allocation d. core/satellite allocation

1.6. Identify the four general strategies for managing a client's income tax liability.

a. tax avoidance: the use of exclusions to reduce the current income tax liability b. tax reduction: the use of deductions and credits to reduce tax liability c. tax deferral: a postponement of the ultimate taxes that must be paid on the income d. income shifting: the transfer of income-producing assets to another taxable entity, typically another individual in a lower marginal income tax bracket than the transferor

List the eight elements that should always be contained in an investment policy statement.

a. the rate of return requirement b. a statement of the risk level acceptable to the client, as well as how risk will be managed or controlled c. any liquidity constraints d. short, intermediate, or long-term horizons should be indicated for the achievement of investment goals e. specific laws and regulations that could impact the investment process f. minimizing income tax and estate tax g. unique preferences and circumstances h. note permitted, or at least, excluded investments

taxable income

adjusted gross income reduced by the greater of the standard deduction or itemized deductions

taxation of return of capital dividend

distribution is generally not taxable. The return of capital reduces the basis in the stock. When, and if, the basis is reduced to zero, any further return of capital dividend is taxed as a capital gain.

investment counsel clientele

have high levels of risk aversion and anxiety; fear not having control over their investment situations; need to feel they are in the driver's seat investment professional: be thorough in evaluating their particular needs; be as careful with their hard-earned money as they are; and offer sincere sympathy with their concerns about risk

Describe the following technical indicators used by contrarians... -short selling -specialists sentiment -mutual fund cash positions -investment advisory opinions -put/call ratio

short selling - Increasing short selling in a stock indicates a growing herd consensus that the market will decline; the contrarian should consider an opposite move. specialists sentiment - Considered to be the "smart money," increased bullishness by specialists in a stock makes contrarians bullish as well, and vice versa. mutual fund cash positions - Mutual fund managers are considered to be herd animals; when mutual funds have low and decreasing cash positions, contrarians take this as a sell signal; the reverse is true as well. investment advisory opinions - This theory suggests that the aggregate opinion of investment advisers is wrong. That is, when the majority of investment advisers are bullish (bearish), a contrarian should sell (buy) securities. put/call ratio - High and increasing put-call ratios suggest buying opportunities to the contrarian, and low or decreasing put-call ratios suggest the opposite.

Tax calculation process...

total income - adjustments to income = AGI - standard deduction or total itemized deductions = taxable income calculate tax - credits + other taxes = net tax liability

3.17. A growth company paid a dividend of $0.50 per share in the last fiscal year. They estimate increasing the dividend by 15% per year for the next three years, after which it is expected to grow at a constant rate of 8%. The investor's required return is 12%. What is the intrinsic value of the stock?

year 1,2,3 dividend - yr 3 is 0.7604 calculate intrinsic value - 0.7604(1.08)/.12-.08=20.53 calculator key strokes = 16.19


Kaugnay na mga set ng pag-aaral

Corporate Social Responsibility - Rutgers - Test 1

View Set

Anatomy & Physiology Lecture - Chapter 8

View Set

Chapter 5: Introduction to Risk, Return, and the Historical Record (Review Questions)

View Set

NMNC 1110 EAQ 7: Fluid and Electrolytes

View Set